Difference between revisions of "2017 AMC 10B Problems/Problem 25"
(Blanked the page) |
|||
Line 1: | Line 1: | ||
+ | ==Cheap Solution== | ||
+ | By inspection, the sequences <math>91,93,92,96,98,100,95</math> and <math>93,91,92,96,98,100,95</math> work, so the answer is $\boxed{\textbf{E } 100}. |
Revision as of 08:57, 16 February 2017
Cheap Solution
By inspection, the sequences and work, so the answer is $\boxed{\textbf{E } 100}.